Вращающийся диск с заряженным краем между специальной и общей теорией относительности

введите описание изображения здесь

Предположим, у нас есть вращающийся диск (вращающийся с постоянной угловой скоростью w относительно инерциального наблюдателя A). Край диска заряжен равномерно общим зарядом Q. Несомненно, А может обнаружить петли магнитного поля с помощью своего карманного компаса благодаря петле постоянного электрического тока, создаваемой заряженным краем вращающегося диска. A наблюдает ток = Q / T = Q w / 2 Pi

Мой вопрос касается наблюдателя B в той же плоскости диска, который вращается вокруг оси диска с той же угловой скоростью w относительно A.

Сможет ли Б обнаружить магнитное поле с помощью своего карманного компаса?

Если мы посмотрим на проблему с точки зрения специальной теории относительности, край заряженного диска движется относительно B (та же омега, но на другом расстоянии от оси диска, то есть относительной скорости), поэтому ее компас может обнаружить магнитное поле.

Но что, если бы В считала себя, согласно ОТО, постоянной относительно всех точек диска, включая край диска, учитывая, что она ожидала бы центробежных и кориолисовых полей? Она должна думать, что ее компас не движется относительно края диска, поэтому компас не должен обнаруживать магнитное поле!

Она правильная или неправильная?

Что такое «постоянная» относительно плоскости диска?
@Cryo Observer B и заряды, распределенные по краю плана диска
Мне очень жаль быть придирчивым, но это трудно понять. Я предполагаю, что заряды распределены, а наблюдатель Б — нет. Заряды распределяются по краю диска. А еще есть «план», который, вероятно, является самолетом. Так вы говорите «Наблюдатель Б и заряды (распределенные по краю диска)»? Какова роль плана/плоскости в этом предложении? Находится ли наблюдатель B в той же плоскости, что и диск? Почему наблюдатель B «постоянен» и не вращается? Если нет, то чем наблюдатель Б отличается от наблюдателя А? Пожалуйста, нарисуйте схему, очень трудно понять, что вы имеете в виду
@Cryo Забудь о самолете. Представьте, что мы живем на планете, подобной Земле, которая вращается вокруг себя, но не имеет собственного магнитного поля. Мы похоронили гонщиков под своим экватором. Каждый заряд просто закапывается под землю на экваторе. Если инерционный наблюдатель А находится где-то не на планете, он увидит, что экватор вращается → петля электрического тока, причина скрытых зарядов на экваторе, следовательно, А обнаруживает магнитное поле. Теперь вопрос, а как же мы? Если мы находимся в наших домах на этой планете и заряды фиксированы относительно нас, то тока нет. Обнаружим ли мы какое-либо магнитное поле?
Не могу сказать без расчетов. Я думаю, что неинерционный наблюдатель обнаружит магнитное поле, но это догадка. Возможно, было бы лучше упростить задачу. Кольцо вращающегося заряда в начале координат, выровненное по оси Z, наблюдатель B в плоскости XY вращается с той же угловой скоростью, что и кольцо. Я бы предложил сначала найти электрическое и магнитное поле для стационарного наблюдателя, а затем проверить, Б 2 Е 2 / с 2 > 0 (см. en.wikipedia.org/wiki/… ). Если это так, то все наблюдатели будут наблюдать какое-то магнитное поле,...
если нет, то нужно больше работы
Примете ли вы вместо этого ответ, связанный с вращающейся сферической оболочкой? Поля инерциальной системы отсчета для этого получить гораздо проще, что в принципе делает возможным точное вычисление.
@MichaelSeifert, да без проблем. Та же идея задачи при условии, что B будет вращаться относительно инерциального наблюдателя A с тем же вектором угловой скорости w заряженной сферы.

Ответы (2)

Я начал гуглить основные результаты для вращающейся сферической оболочки и обнаружил, что кто-то опередил меня в этом самом расчете:

Электромагнитные поля вращающейся заряженной оболочки , Кирк Т. Макдональд

В статье автор вычисляет поля в инерциальной системе отсчета, а затем обеспечивает преобразование во вращающуюся систему, чтобы найти магнитное и электрическое поля, наблюдаемые в этой системе. Автор рассматривает более общий случай, когда снаряд с зарядом Вопрос и радиус а вращается с угловой скоростью ю ю , а вращающаяся рамка вращается на ю ю ; случай, запрошенный ОП, просто случай ю ю "=" ю ю .

Результат внутри оболочки на самом деле удивительно прост: в пределе медленных вращений магнитное поле, измеренное коротирующим наблюдателем внутри оболочки, точно такое же, как магнитное поле, измеренное инерционным наблюдателем внутри оболочки. Причина этого довольно проста: преобразование между полями для двух инерциальных систем отсчета в пределе малых скоростей равно

Б Б в с Е .
Внутри оболочки, Е "=" 0 и так Б "=" Б . Таким образом, если инерционный наблюдатель измеряет магнитное поле внутри оболочки, то же самое будет и с коротирующим наблюдателем.

Причина, по которой внутри оболочки все еще существует поле, даже если вращающийся наблюдатель не видит токов, заключается просто в том, что уравнения Максвелла не выполняются во вращающейся системе отсчета. В приведенной выше статье представлены уравнения Максвелла в (медленно) вращающейся системе отсчета в уравнениях. (31–34) приведенных выше заметок. В отсутствие связанных источников и в стационарном случае (как здесь) они становятся

Е "=" 4 π [ р в с 2 Дж + ю ю Б 2 π с ] , × Б "=" 4 π с [ Дж + в р + ю ю × Е 4 π ю 4 π Е ф ] ,
где в "=" ю ю × р и ф (Два других уравнения Максвелла остаются в силе; в примечаниях ю ю вместо ю ю , но я считаю, что это ошибка.) Во вращающейся рамке, в то время как Дж "=" 0 , в магнитное поле по-прежнему вносятся вклады от других членов правой части последнего уравнения, и, таким образом, магнитное поле не обращается в нуль. (Я полагаю, что последние два члена в данном случае сокращаются, так как они соответствуют скорости изменения Е в отношении т ; но р в термин, конечно, нет.)

Ссылки в связанной статье также могут представлять интерес; в частности, вы можете обратиться к заметкам того же автора по электродинамике во вращающихся системах отсчета , а также к следующим статьям:

  • Л. И. Шифф, "Вопрос общей теории относительности", Proc. Нац. акад. науч. 25, 391 (1939)
  • CT Ridgely, "Применение релятивистской электродинамики к вращающейся материальной среде", Am. Дж. Физ. 66, 114 (1998)
  • CT Ridgely, "Применение ковариантных и контравариантных электромагнитных тензоров к вращающимся средам", Am. Дж. Физ. 67, 414 (1999)
Моя интуиция подсказывает, что это остаточное магнитное поле обусловлено уравнениями Максвелла, не горячими в неинерциальных системах отсчета. Я твердо уверен, что движущийся наблюдатель не должен замечать никакого магнитного поля в общем случае, поскольку с точки зрения ОТО нет никакого «относительного тока». Также я не понял утверждения автора "в той системе отсчёта наблюдается ненулевое магнитное поле, что можно списать на "фиктивные" токи. В этой системе наблюдается ненулевое магнитное поле, что можно отнести к " фиктивные «токи». что такое «фиктивный» ток?!
@AhmedKamalKassem: я полагаю, что это относится к дополнительным терминам, которые появляются в правой части закона Гаусса и закона Ампера, которые я написал выше (для особого случая). Аналогия с «фиктивными силами», которые появляются, когда Законы Ньютона записаны в неинерциальной системе отсчета. Автор заметок, на которые я ссылался, похоже, получил терминологию из статьи Шиффа, на которую я ссылаюсь в конце ответа.

Итак, я выработаю ответ для вращающегося кольца заряда радиуса р , которую можно описать плотностью тока:

Дж "=" я 0 2 π р . г ф ( грех ф потому что ф 0 ) дельта ( 3 ) ( р р ( потому что ф грех ф 0 ) )

Где я ток в кольце. Это для инерциального наблюдателя и в декартовых координатах. Это можно использовать в законе Био-Савара , чтобы найти магнитное поле ( мю 0 вакуумная проницаемость):

Б ( р ) "=" мю 0 4 π г 3 р Дж ( р ) × ( р р ) | р р | 3

Для наблюдателя (А), сидящего в плоскости XY, на расстоянии р вдали от начала координат магнитное поле будет z-поляризованным:

Б г ( р ) "=" я мю 0 4 π 0 2 π р г ф . р р потому что ф ( р 2 + р 2 2 р р потому что ф ) 3 / 2 "=" я мю 0 4 π р К ( р , р )

Где

К ( р , р ) "=" 0 2 π г ф . р 2 ( р р потому что ф ) ( р 2 + р 2 2 р р потому что ф ) 3 / 2

это просто сокращение для безразмерного интеграла, который я не знаю, как вычислить (кроме как в пределе или численно).

Пусть полный заряд петли будет Вопрос , пусть угловая скорость вращения ю . Так я "=" Вопрос ю / 2 π и:

Б г ( р ) "=" Вопрос ю мю 0 8 π 2 р К ( р , р )

Далее электрическое поле от кольца плотности заряда:

р "=" Вопрос 2 π 0 2 π г ф дельта ( 3 ) ( р р ( потому что ф грех ф 0 ) )

Для наблюдателя А электрическое поле будет чисто радиальным ( ϵ 0 диэлектрическая проницаемость вакуума):

Е р ( р ) "=" Вопрос / 2 π 4 π ϵ 0 0 2 π г ф . р р потому что ф ( р 2 + р 2 2 р р потому что ф ) 3 / 2

снова представляем:

С ( р , р ) "=" 0 2 π г ф . р 2 ( р р потому что ф ) ( р 2 + р 2 2 р р потому что ф ) 3 / 2

электрическое поле становится

Е р ( р ) "=" Вопрос 8 π 2 р 2 ϵ 0 С ( р , р )

Теперь рассмотрим:

Б г Е р / с "=" ю р с × К ( р , р ) С ( р , р )

Где с это скорость света. Играть с р р показывает, что для К / С 1 , поэтому ключевая величина здесь ю р / с которая представляет собой отношение скорости, с которой заряды движутся в петле, к скорости света.

Четко ю р / с < 1 , но для больших угловых скоростей в принципе можно иметь с Б г / Е р > 1 . Тогда все наблюдатели, проходящие через эту точку пространства, будут наблюдать ненулевое магнитное поле. Логика основана на Б 2 Е 2 / с 2 инвариант электромагнитного поля

При нормальных настройках электрические и магнитные поля, видимые инерциальным наблюдателем, будут, если предположить, что наблюдатель смещен от начала координат по оси x:

Е Икс "=" Е Б г ю р с Е

Отсюда вы сможете применить преобразования Лоренца для аппроксимации электромагнитного поля в мгновенной инерциальной системе отсчета вращающегося наблюдателя B.


Заметил такое соотношение К / С расходится для р р , поэтому, если наблюдатель сидит в петле, у вас, вероятно, может возникнуть ситуация с Б г / Е р > 1 даже при относительно умеренных скоростях вращения